$a,b,c>0$ and $abc=1$; prove $sum_{cyc}frac1{(b+1)^2}+frac1{a+b+c+1}ge1$ [duplicate] The...

Proper way to express "He disappeared them"

Grabbing quick drinks

Would a grinding machine be a simple and workable propulsion system for an interplanetary spacecraft?

How to count occurrences of text in a file?

Is the D&D universe the same as the Forgotten Realms universe?

What did we know about the Kessel run before the prequels?

Why isn't acceleration always zero whenever velocity is zero, such as the moment a ball bounces off a wall?

How to write a definition with variants?

Is French Guiana a (hard) EU border?

Do I need to write [sic] when a number is less than 10 but isn't written out?

Can you be charged for obstruction for refusing to answer questions?

Why don't programming languages automatically manage the synchronous/asynchronous problem?

Axiom Schema vs Axiom

Is it ever safe to open a suspicious HTML file (e.g. email attachment)?

The exact meaning of 'Mom made me a sandwich'

What connection does MS Office have to Netscape Navigator?

Math-accent symbol over parentheses enclosing accented symbol (amsmath)

Why is the US ranked as #45 in Press Freedom ratings, despite its extremely permissive free speech laws?

Why didn't Khan get resurrected in the Genesis Explosion?

What steps are necessary to read a Modern SSD in Medieval Europe?

How to prove a simple equation?

0 rank tensor vs 1D vector

Why does standard notation not preserve intervals (visually)

The past simple of "gaslight" – "gaslighted" or "gaslit"?



$a,b,c>0$ and $abc=1$; prove $sum_{cyc}frac1{(b+1)^2}+frac1{a+b+c+1}ge1$ [duplicate]



The Next CEO of Stack OverflowProve $Σ_{cyc}frac{1}{left(a+1right)^2}+frac{1}{a+b+c+1}ge 1$Prove that $sumlimits_{cyc}frac{a}{b(3+a-b)}ge 1$For what values of $k>0$ does $abc=1 implies sum_{mbox{cyc}}left(frac1{a+k}-frac{a}{a^2+k}right) geq 0$?show $sum_{cyc}(1-x)^2ge sum_{cyc}frac{z^2(1-x^2)(1-y^2)}{(xy+z)^2}.$prove $sum_{cyc}frac{1}{a(a+b)}gefrac{4}{ac+bd}$Inequality $sum_{cyc} sqrt{frac{a}{a+8}} geq 1$prove this inequality $Σ_{cyc}frac{a^3+abc}{b^2+c^2}ge a+b+c$For $abc=1$ prove that $sumlimits_{cyc}frac{a}{a^{11}+1}leqfrac{3}{2}.$prove this inequality by $abc=1$If $ab+bc+ca=3$ for non-negative $a$, $b$, $c$, show that $sum_{cyc}a^2b^2+sum_{cyc}frac{12a^2b^2c^2}{(a+b)^2}ge 12abc$Prove the inequality $sum_{cyc} {{a+abc} over {1+ab+abcd}} ge {{10} over {3}}$ with Cauchy-Schwarz












0












$begingroup$



This question already has an answer here:




  • Prove $Σ_{cyc}frac{1}{left(a+1right)^2}+frac{1}{a+b+c+1}ge 1$

    1 answer





Let $a$, $b$ and $c$ be three positives such that $abc=1$. Prove that $$sum_{cyc}frac1{(b+1)^2}+frac1{a+b+c+1}ge1$$




Here's what have I done that is completely incorrect. Let $a + b + c + 1 = x$. We have that
$$displaystyle sum_{cyc}dfrac{1}{(b + 1)^2} ge displaystyle sum_{cyc}dfrac{1}{(c + 1)(a + 1)} = sum_{cyc}dfrac{1}{x - b + ca}$$
That means
$$displaystyle sum_{cyc}dfrac{1}{(b + 1)^2} + dfrac{1}{a + b + c + 1} ge dfrac{16}{bc + ca + ab + 3x + 1}$$
$$ge dfrac{48}{(x - 1)^2 + 9x + 3} = dfrac{48}{x^2 + 7x + 4}$$
That was illogical.










share|cite|improve this question











$endgroup$



marked as duplicate by Carl Mummert, Michael Rozenberg inequality
Users with the  inequality badge can single-handedly close inequality questions as duplicates and reopen them as needed.

StackExchange.ready(function() {
if (StackExchange.options.isMobile) return;

$('.dupe-hammer-message-hover:not(.hover-bound)').each(function() {
var $hover = $(this).addClass('hover-bound'),
$msg = $hover.siblings('.dupe-hammer-message');

$hover.hover(
function() {
$hover.showInfoMessage('', {
messageElement: $msg.clone().show(),
transient: false,
position: { my: 'bottom left', at: 'top center', offsetTop: -7 },
dismissable: false,
relativeToBody: true
});
},
function() {
StackExchange.helpers.removeMessages();
}
);
});
});
Mar 17 at 12:06


This question has been asked before and already has an answer. If those answers do not fully address your question, please ask a new question.














  • 1




    $begingroup$
    Title is very odd.
    $endgroup$
    – Love Invariants
    Mar 16 at 17:33
















0












$begingroup$



This question already has an answer here:




  • Prove $Σ_{cyc}frac{1}{left(a+1right)^2}+frac{1}{a+b+c+1}ge 1$

    1 answer





Let $a$, $b$ and $c$ be three positives such that $abc=1$. Prove that $$sum_{cyc}frac1{(b+1)^2}+frac1{a+b+c+1}ge1$$




Here's what have I done that is completely incorrect. Let $a + b + c + 1 = x$. We have that
$$displaystyle sum_{cyc}dfrac{1}{(b + 1)^2} ge displaystyle sum_{cyc}dfrac{1}{(c + 1)(a + 1)} = sum_{cyc}dfrac{1}{x - b + ca}$$
That means
$$displaystyle sum_{cyc}dfrac{1}{(b + 1)^2} + dfrac{1}{a + b + c + 1} ge dfrac{16}{bc + ca + ab + 3x + 1}$$
$$ge dfrac{48}{(x - 1)^2 + 9x + 3} = dfrac{48}{x^2 + 7x + 4}$$
That was illogical.










share|cite|improve this question











$endgroup$



marked as duplicate by Carl Mummert, Michael Rozenberg inequality
Users with the  inequality badge can single-handedly close inequality questions as duplicates and reopen them as needed.

StackExchange.ready(function() {
if (StackExchange.options.isMobile) return;

$('.dupe-hammer-message-hover:not(.hover-bound)').each(function() {
var $hover = $(this).addClass('hover-bound'),
$msg = $hover.siblings('.dupe-hammer-message');

$hover.hover(
function() {
$hover.showInfoMessage('', {
messageElement: $msg.clone().show(),
transient: false,
position: { my: 'bottom left', at: 'top center', offsetTop: -7 },
dismissable: false,
relativeToBody: true
});
},
function() {
StackExchange.helpers.removeMessages();
}
);
});
});
Mar 17 at 12:06


This question has been asked before and already has an answer. If those answers do not fully address your question, please ask a new question.














  • 1




    $begingroup$
    Title is very odd.
    $endgroup$
    – Love Invariants
    Mar 16 at 17:33














0












0








0


1



$begingroup$



This question already has an answer here:




  • Prove $Σ_{cyc}frac{1}{left(a+1right)^2}+frac{1}{a+b+c+1}ge 1$

    1 answer





Let $a$, $b$ and $c$ be three positives such that $abc=1$. Prove that $$sum_{cyc}frac1{(b+1)^2}+frac1{a+b+c+1}ge1$$




Here's what have I done that is completely incorrect. Let $a + b + c + 1 = x$. We have that
$$displaystyle sum_{cyc}dfrac{1}{(b + 1)^2} ge displaystyle sum_{cyc}dfrac{1}{(c + 1)(a + 1)} = sum_{cyc}dfrac{1}{x - b + ca}$$
That means
$$displaystyle sum_{cyc}dfrac{1}{(b + 1)^2} + dfrac{1}{a + b + c + 1} ge dfrac{16}{bc + ca + ab + 3x + 1}$$
$$ge dfrac{48}{(x - 1)^2 + 9x + 3} = dfrac{48}{x^2 + 7x + 4}$$
That was illogical.










share|cite|improve this question











$endgroup$





This question already has an answer here:




  • Prove $Σ_{cyc}frac{1}{left(a+1right)^2}+frac{1}{a+b+c+1}ge 1$

    1 answer





Let $a$, $b$ and $c$ be three positives such that $abc=1$. Prove that $$sum_{cyc}frac1{(b+1)^2}+frac1{a+b+c+1}ge1$$




Here's what have I done that is completely incorrect. Let $a + b + c + 1 = x$. We have that
$$displaystyle sum_{cyc}dfrac{1}{(b + 1)^2} ge displaystyle sum_{cyc}dfrac{1}{(c + 1)(a + 1)} = sum_{cyc}dfrac{1}{x - b + ca}$$
That means
$$displaystyle sum_{cyc}dfrac{1}{(b + 1)^2} + dfrac{1}{a + b + c + 1} ge dfrac{16}{bc + ca + ab + 3x + 1}$$
$$ge dfrac{48}{(x - 1)^2 + 9x + 3} = dfrac{48}{x^2 + 7x + 4}$$
That was illogical.





This question already has an answer here:




  • Prove $Σ_{cyc}frac{1}{left(a+1right)^2}+frac{1}{a+b+c+1}ge 1$

    1 answer








algebra-precalculus inequality substitution






share|cite|improve this question















share|cite|improve this question













share|cite|improve this question




share|cite|improve this question








edited Mar 17 at 5:15









Michael Rozenberg

109k1896201




109k1896201










asked Mar 16 at 17:13









Lê Thành ĐạtLê Thành Đạt

33312




33312




marked as duplicate by Carl Mummert, Michael Rozenberg inequality
Users with the  inequality badge can single-handedly close inequality questions as duplicates and reopen them as needed.

StackExchange.ready(function() {
if (StackExchange.options.isMobile) return;

$('.dupe-hammer-message-hover:not(.hover-bound)').each(function() {
var $hover = $(this).addClass('hover-bound'),
$msg = $hover.siblings('.dupe-hammer-message');

$hover.hover(
function() {
$hover.showInfoMessage('', {
messageElement: $msg.clone().show(),
transient: false,
position: { my: 'bottom left', at: 'top center', offsetTop: -7 },
dismissable: false,
relativeToBody: true
});
},
function() {
StackExchange.helpers.removeMessages();
}
);
});
});
Mar 17 at 12:06


This question has been asked before and already has an answer. If those answers do not fully address your question, please ask a new question.









marked as duplicate by Carl Mummert, Michael Rozenberg inequality
Users with the  inequality badge can single-handedly close inequality questions as duplicates and reopen them as needed.

StackExchange.ready(function() {
if (StackExchange.options.isMobile) return;

$('.dupe-hammer-message-hover:not(.hover-bound)').each(function() {
var $hover = $(this).addClass('hover-bound'),
$msg = $hover.siblings('.dupe-hammer-message');

$hover.hover(
function() {
$hover.showInfoMessage('', {
messageElement: $msg.clone().show(),
transient: false,
position: { my: 'bottom left', at: 'top center', offsetTop: -7 },
dismissable: false,
relativeToBody: true
});
},
function() {
StackExchange.helpers.removeMessages();
}
);
});
});
Mar 17 at 12:06


This question has been asked before and already has an answer. If those answers do not fully address your question, please ask a new question.










  • 1




    $begingroup$
    Title is very odd.
    $endgroup$
    – Love Invariants
    Mar 16 at 17:33














  • 1




    $begingroup$
    Title is very odd.
    $endgroup$
    – Love Invariants
    Mar 16 at 17:33








1




1




$begingroup$
Title is very odd.
$endgroup$
– Love Invariants
Mar 16 at 17:33




$begingroup$
Title is very odd.
$endgroup$
– Love Invariants
Mar 16 at 17:33










3 Answers
3






active

oldest

votes


















1












$begingroup$

Let $a+b+c=3u$, $ab+ac+bc=3v^2,$ where $v>0$, and $abc=w^3$.



Thus, $$sum_{cyc}frac{1}{(a+1)^2}+frac{1}{a+b+c+1}-1=$$
$$=sum_{cyc}frac{1}{(a+1)^2}+frac{2}{prodlimits_{cyc}(a+1)}-1+frac{1}{a+b+c+1}-frac{2}{prodlimits_{cyc}(a+1)}=$$
$$=frac{a^2+b^2+c^2-3}{prodlimits_{cyc}(a+1)^2}+frac{sumlimits_{cyc}(ab-a)}{(a+b+c+1)prodlimits_{cyc}(a+1)}.$$
Thus, we need to prove that
$$(a^2+b^2+c^2-3)(a+b+c+1)+sum_{cyc}(ab-a)prod_{cyc}(a+1)geq0$$ or
$$3v^4+9u^3w-6uv^2w-5uw^3-w^4geq0,$$ which is true because $ugeq vgeq w$.






share|cite|improve this answer









$endgroup$













  • $begingroup$
    Could you fix your answer if possible because the given condition is already $abc = 1$. There's no need to let $abc = w^3$.
    $endgroup$
    – Lê Thành Đạt
    Mar 17 at 0:44










  • $begingroup$
    @Lê Thành Đạt Just $w=1$, which gives a homogenization. See please better my solution.
    $endgroup$
    – Michael Rozenberg
    Mar 17 at 2:15










  • $begingroup$
    Is there a way to prove the inequation without using the $uvw$'s method and by using classic inequalities?
    $endgroup$
    – Lê Thành Đạt
    Mar 17 at 2:58










  • $begingroup$
    @Lê Thành Đạt There is a very nice proof. But it's not mine.
    $endgroup$
    – Michael Rozenberg
    Mar 17 at 4:15










  • $begingroup$
    Please post the proof if it is not the other answer.
    $endgroup$
    – Lê Thành Đạt
    Mar 17 at 4:18



















2












$begingroup$

Hint: Substituting $$a=frac{x}{y},b=frac{y}{z}c=frac{z}{x}$$
so we get



$${x}^{6}{z}^{3}+{x}^{5}{y}^{3}z+{x}^{5}{y}^{2}{z}^{2}+{x}^{4}{y}^{4}z-3
,{x}^{4}{y}^{2}{z}^{3}+{x}^{4}y{z}^{4}+{x}^{3}{y}^{6}-3,{x}^{3}{y}^{
4}{z}^{2}-3,{x}^{3}{y}^{3}{z}^{3}+{x}^{3}y{z}^{5}+{x}^{2}{y}^{5}{z}^{
2}-3,{x}^{2}{y}^{3}{z}^{4}+{x}^{2}{y}^{2}{z}^{5}+x{y}^{5}{z}^{3}+x{y}
^{4}{z}^{4}+{y}^{3}{z}^{6}
geq 0$$

This is true by AM-GM.






share|cite|improve this answer









$endgroup$













  • $begingroup$
    WA says the same thing: wolframalpha.com/input/…
    $endgroup$
    – Michael Rozenberg
    Mar 16 at 19:12










  • $begingroup$
    Should i misreaded the problem?
    $endgroup$
    – Dr. Sonnhard Graubner
    Mar 16 at 19:13










  • $begingroup$
    I can't get around proving the inequality with this method. Do you have a different answer?
    $endgroup$
    – Lê Thành Đạt
    Mar 17 at 11:27



















1












$begingroup$

Another solution.



Since $$prod_{cyc}(a-1)^2=prod_{cyc}((a-1)(b-1))geq0,$$ we can assume that $(a-1)(b-1)geq0,$ which gives
$$a+bleq ab+1=frac{1}{c}+1.$$



Also, we have $$frac{1}{(a+1)^2}+frac{1}{(b+1)^2}-frac{1}{ab+1}=frac{ab(a-b)^2+(ab-1)^2}{(a+1)^2(b+1)^2}geq0$$
Id est,
$$sum_{cyc}frac{1}{(a+1)^2}+frac{1}{a+b+c+1}geqfrac{1}{ab+1}+frac{1}{(c+1)^2}+frac{1}{ab+1+c+1}=$$
$$=frac{1}{frac{1}{c}+1}+frac{1}{(c+1)^2}+frac{1}{frac{1}{c}+1+c+1}=frac{c}{c+1}+frac{1}{(c+1)^2}+frac{c}{(c+1)^2}=1.$$






share|cite|improve this answer









$endgroup$




















    3 Answers
    3






    active

    oldest

    votes








    3 Answers
    3






    active

    oldest

    votes









    active

    oldest

    votes






    active

    oldest

    votes









    1












    $begingroup$

    Let $a+b+c=3u$, $ab+ac+bc=3v^2,$ where $v>0$, and $abc=w^3$.



    Thus, $$sum_{cyc}frac{1}{(a+1)^2}+frac{1}{a+b+c+1}-1=$$
    $$=sum_{cyc}frac{1}{(a+1)^2}+frac{2}{prodlimits_{cyc}(a+1)}-1+frac{1}{a+b+c+1}-frac{2}{prodlimits_{cyc}(a+1)}=$$
    $$=frac{a^2+b^2+c^2-3}{prodlimits_{cyc}(a+1)^2}+frac{sumlimits_{cyc}(ab-a)}{(a+b+c+1)prodlimits_{cyc}(a+1)}.$$
    Thus, we need to prove that
    $$(a^2+b^2+c^2-3)(a+b+c+1)+sum_{cyc}(ab-a)prod_{cyc}(a+1)geq0$$ or
    $$3v^4+9u^3w-6uv^2w-5uw^3-w^4geq0,$$ which is true because $ugeq vgeq w$.






    share|cite|improve this answer









    $endgroup$













    • $begingroup$
      Could you fix your answer if possible because the given condition is already $abc = 1$. There's no need to let $abc = w^3$.
      $endgroup$
      – Lê Thành Đạt
      Mar 17 at 0:44










    • $begingroup$
      @Lê Thành Đạt Just $w=1$, which gives a homogenization. See please better my solution.
      $endgroup$
      – Michael Rozenberg
      Mar 17 at 2:15










    • $begingroup$
      Is there a way to prove the inequation without using the $uvw$'s method and by using classic inequalities?
      $endgroup$
      – Lê Thành Đạt
      Mar 17 at 2:58










    • $begingroup$
      @Lê Thành Đạt There is a very nice proof. But it's not mine.
      $endgroup$
      – Michael Rozenberg
      Mar 17 at 4:15










    • $begingroup$
      Please post the proof if it is not the other answer.
      $endgroup$
      – Lê Thành Đạt
      Mar 17 at 4:18
















    1












    $begingroup$

    Let $a+b+c=3u$, $ab+ac+bc=3v^2,$ where $v>0$, and $abc=w^3$.



    Thus, $$sum_{cyc}frac{1}{(a+1)^2}+frac{1}{a+b+c+1}-1=$$
    $$=sum_{cyc}frac{1}{(a+1)^2}+frac{2}{prodlimits_{cyc}(a+1)}-1+frac{1}{a+b+c+1}-frac{2}{prodlimits_{cyc}(a+1)}=$$
    $$=frac{a^2+b^2+c^2-3}{prodlimits_{cyc}(a+1)^2}+frac{sumlimits_{cyc}(ab-a)}{(a+b+c+1)prodlimits_{cyc}(a+1)}.$$
    Thus, we need to prove that
    $$(a^2+b^2+c^2-3)(a+b+c+1)+sum_{cyc}(ab-a)prod_{cyc}(a+1)geq0$$ or
    $$3v^4+9u^3w-6uv^2w-5uw^3-w^4geq0,$$ which is true because $ugeq vgeq w$.






    share|cite|improve this answer









    $endgroup$













    • $begingroup$
      Could you fix your answer if possible because the given condition is already $abc = 1$. There's no need to let $abc = w^3$.
      $endgroup$
      – Lê Thành Đạt
      Mar 17 at 0:44










    • $begingroup$
      @Lê Thành Đạt Just $w=1$, which gives a homogenization. See please better my solution.
      $endgroup$
      – Michael Rozenberg
      Mar 17 at 2:15










    • $begingroup$
      Is there a way to prove the inequation without using the $uvw$'s method and by using classic inequalities?
      $endgroup$
      – Lê Thành Đạt
      Mar 17 at 2:58










    • $begingroup$
      @Lê Thành Đạt There is a very nice proof. But it's not mine.
      $endgroup$
      – Michael Rozenberg
      Mar 17 at 4:15










    • $begingroup$
      Please post the proof if it is not the other answer.
      $endgroup$
      – Lê Thành Đạt
      Mar 17 at 4:18














    1












    1








    1





    $begingroup$

    Let $a+b+c=3u$, $ab+ac+bc=3v^2,$ where $v>0$, and $abc=w^3$.



    Thus, $$sum_{cyc}frac{1}{(a+1)^2}+frac{1}{a+b+c+1}-1=$$
    $$=sum_{cyc}frac{1}{(a+1)^2}+frac{2}{prodlimits_{cyc}(a+1)}-1+frac{1}{a+b+c+1}-frac{2}{prodlimits_{cyc}(a+1)}=$$
    $$=frac{a^2+b^2+c^2-3}{prodlimits_{cyc}(a+1)^2}+frac{sumlimits_{cyc}(ab-a)}{(a+b+c+1)prodlimits_{cyc}(a+1)}.$$
    Thus, we need to prove that
    $$(a^2+b^2+c^2-3)(a+b+c+1)+sum_{cyc}(ab-a)prod_{cyc}(a+1)geq0$$ or
    $$3v^4+9u^3w-6uv^2w-5uw^3-w^4geq0,$$ which is true because $ugeq vgeq w$.






    share|cite|improve this answer









    $endgroup$



    Let $a+b+c=3u$, $ab+ac+bc=3v^2,$ where $v>0$, and $abc=w^3$.



    Thus, $$sum_{cyc}frac{1}{(a+1)^2}+frac{1}{a+b+c+1}-1=$$
    $$=sum_{cyc}frac{1}{(a+1)^2}+frac{2}{prodlimits_{cyc}(a+1)}-1+frac{1}{a+b+c+1}-frac{2}{prodlimits_{cyc}(a+1)}=$$
    $$=frac{a^2+b^2+c^2-3}{prodlimits_{cyc}(a+1)^2}+frac{sumlimits_{cyc}(ab-a)}{(a+b+c+1)prodlimits_{cyc}(a+1)}.$$
    Thus, we need to prove that
    $$(a^2+b^2+c^2-3)(a+b+c+1)+sum_{cyc}(ab-a)prod_{cyc}(a+1)geq0$$ or
    $$3v^4+9u^3w-6uv^2w-5uw^3-w^4geq0,$$ which is true because $ugeq vgeq w$.







    share|cite|improve this answer












    share|cite|improve this answer



    share|cite|improve this answer










    answered Mar 16 at 18:17









    Michael RozenbergMichael Rozenberg

    109k1896201




    109k1896201












    • $begingroup$
      Could you fix your answer if possible because the given condition is already $abc = 1$. There's no need to let $abc = w^3$.
      $endgroup$
      – Lê Thành Đạt
      Mar 17 at 0:44










    • $begingroup$
      @Lê Thành Đạt Just $w=1$, which gives a homogenization. See please better my solution.
      $endgroup$
      – Michael Rozenberg
      Mar 17 at 2:15










    • $begingroup$
      Is there a way to prove the inequation without using the $uvw$'s method and by using classic inequalities?
      $endgroup$
      – Lê Thành Đạt
      Mar 17 at 2:58










    • $begingroup$
      @Lê Thành Đạt There is a very nice proof. But it's not mine.
      $endgroup$
      – Michael Rozenberg
      Mar 17 at 4:15










    • $begingroup$
      Please post the proof if it is not the other answer.
      $endgroup$
      – Lê Thành Đạt
      Mar 17 at 4:18


















    • $begingroup$
      Could you fix your answer if possible because the given condition is already $abc = 1$. There's no need to let $abc = w^3$.
      $endgroup$
      – Lê Thành Đạt
      Mar 17 at 0:44










    • $begingroup$
      @Lê Thành Đạt Just $w=1$, which gives a homogenization. See please better my solution.
      $endgroup$
      – Michael Rozenberg
      Mar 17 at 2:15










    • $begingroup$
      Is there a way to prove the inequation without using the $uvw$'s method and by using classic inequalities?
      $endgroup$
      – Lê Thành Đạt
      Mar 17 at 2:58










    • $begingroup$
      @Lê Thành Đạt There is a very nice proof. But it's not mine.
      $endgroup$
      – Michael Rozenberg
      Mar 17 at 4:15










    • $begingroup$
      Please post the proof if it is not the other answer.
      $endgroup$
      – Lê Thành Đạt
      Mar 17 at 4:18
















    $begingroup$
    Could you fix your answer if possible because the given condition is already $abc = 1$. There's no need to let $abc = w^3$.
    $endgroup$
    – Lê Thành Đạt
    Mar 17 at 0:44




    $begingroup$
    Could you fix your answer if possible because the given condition is already $abc = 1$. There's no need to let $abc = w^3$.
    $endgroup$
    – Lê Thành Đạt
    Mar 17 at 0:44












    $begingroup$
    @Lê Thành Đạt Just $w=1$, which gives a homogenization. See please better my solution.
    $endgroup$
    – Michael Rozenberg
    Mar 17 at 2:15




    $begingroup$
    @Lê Thành Đạt Just $w=1$, which gives a homogenization. See please better my solution.
    $endgroup$
    – Michael Rozenberg
    Mar 17 at 2:15












    $begingroup$
    Is there a way to prove the inequation without using the $uvw$'s method and by using classic inequalities?
    $endgroup$
    – Lê Thành Đạt
    Mar 17 at 2:58




    $begingroup$
    Is there a way to prove the inequation without using the $uvw$'s method and by using classic inequalities?
    $endgroup$
    – Lê Thành Đạt
    Mar 17 at 2:58












    $begingroup$
    @Lê Thành Đạt There is a very nice proof. But it's not mine.
    $endgroup$
    – Michael Rozenberg
    Mar 17 at 4:15




    $begingroup$
    @Lê Thành Đạt There is a very nice proof. But it's not mine.
    $endgroup$
    – Michael Rozenberg
    Mar 17 at 4:15












    $begingroup$
    Please post the proof if it is not the other answer.
    $endgroup$
    – Lê Thành Đạt
    Mar 17 at 4:18




    $begingroup$
    Please post the proof if it is not the other answer.
    $endgroup$
    – Lê Thành Đạt
    Mar 17 at 4:18











    2












    $begingroup$

    Hint: Substituting $$a=frac{x}{y},b=frac{y}{z}c=frac{z}{x}$$
    so we get



    $${x}^{6}{z}^{3}+{x}^{5}{y}^{3}z+{x}^{5}{y}^{2}{z}^{2}+{x}^{4}{y}^{4}z-3
    ,{x}^{4}{y}^{2}{z}^{3}+{x}^{4}y{z}^{4}+{x}^{3}{y}^{6}-3,{x}^{3}{y}^{
    4}{z}^{2}-3,{x}^{3}{y}^{3}{z}^{3}+{x}^{3}y{z}^{5}+{x}^{2}{y}^{5}{z}^{
    2}-3,{x}^{2}{y}^{3}{z}^{4}+{x}^{2}{y}^{2}{z}^{5}+x{y}^{5}{z}^{3}+x{y}
    ^{4}{z}^{4}+{y}^{3}{z}^{6}
    geq 0$$

    This is true by AM-GM.






    share|cite|improve this answer









    $endgroup$













    • $begingroup$
      WA says the same thing: wolframalpha.com/input/…
      $endgroup$
      – Michael Rozenberg
      Mar 16 at 19:12










    • $begingroup$
      Should i misreaded the problem?
      $endgroup$
      – Dr. Sonnhard Graubner
      Mar 16 at 19:13










    • $begingroup$
      I can't get around proving the inequality with this method. Do you have a different answer?
      $endgroup$
      – Lê Thành Đạt
      Mar 17 at 11:27
















    2












    $begingroup$

    Hint: Substituting $$a=frac{x}{y},b=frac{y}{z}c=frac{z}{x}$$
    so we get



    $${x}^{6}{z}^{3}+{x}^{5}{y}^{3}z+{x}^{5}{y}^{2}{z}^{2}+{x}^{4}{y}^{4}z-3
    ,{x}^{4}{y}^{2}{z}^{3}+{x}^{4}y{z}^{4}+{x}^{3}{y}^{6}-3,{x}^{3}{y}^{
    4}{z}^{2}-3,{x}^{3}{y}^{3}{z}^{3}+{x}^{3}y{z}^{5}+{x}^{2}{y}^{5}{z}^{
    2}-3,{x}^{2}{y}^{3}{z}^{4}+{x}^{2}{y}^{2}{z}^{5}+x{y}^{5}{z}^{3}+x{y}
    ^{4}{z}^{4}+{y}^{3}{z}^{6}
    geq 0$$

    This is true by AM-GM.






    share|cite|improve this answer









    $endgroup$













    • $begingroup$
      WA says the same thing: wolframalpha.com/input/…
      $endgroup$
      – Michael Rozenberg
      Mar 16 at 19:12










    • $begingroup$
      Should i misreaded the problem?
      $endgroup$
      – Dr. Sonnhard Graubner
      Mar 16 at 19:13










    • $begingroup$
      I can't get around proving the inequality with this method. Do you have a different answer?
      $endgroup$
      – Lê Thành Đạt
      Mar 17 at 11:27














    2












    2








    2





    $begingroup$

    Hint: Substituting $$a=frac{x}{y},b=frac{y}{z}c=frac{z}{x}$$
    so we get



    $${x}^{6}{z}^{3}+{x}^{5}{y}^{3}z+{x}^{5}{y}^{2}{z}^{2}+{x}^{4}{y}^{4}z-3
    ,{x}^{4}{y}^{2}{z}^{3}+{x}^{4}y{z}^{4}+{x}^{3}{y}^{6}-3,{x}^{3}{y}^{
    4}{z}^{2}-3,{x}^{3}{y}^{3}{z}^{3}+{x}^{3}y{z}^{5}+{x}^{2}{y}^{5}{z}^{
    2}-3,{x}^{2}{y}^{3}{z}^{4}+{x}^{2}{y}^{2}{z}^{5}+x{y}^{5}{z}^{3}+x{y}
    ^{4}{z}^{4}+{y}^{3}{z}^{6}
    geq 0$$

    This is true by AM-GM.






    share|cite|improve this answer









    $endgroup$



    Hint: Substituting $$a=frac{x}{y},b=frac{y}{z}c=frac{z}{x}$$
    so we get



    $${x}^{6}{z}^{3}+{x}^{5}{y}^{3}z+{x}^{5}{y}^{2}{z}^{2}+{x}^{4}{y}^{4}z-3
    ,{x}^{4}{y}^{2}{z}^{3}+{x}^{4}y{z}^{4}+{x}^{3}{y}^{6}-3,{x}^{3}{y}^{
    4}{z}^{2}-3,{x}^{3}{y}^{3}{z}^{3}+{x}^{3}y{z}^{5}+{x}^{2}{y}^{5}{z}^{
    2}-3,{x}^{2}{y}^{3}{z}^{4}+{x}^{2}{y}^{2}{z}^{5}+x{y}^{5}{z}^{3}+x{y}
    ^{4}{z}^{4}+{y}^{3}{z}^{6}
    geq 0$$

    This is true by AM-GM.







    share|cite|improve this answer












    share|cite|improve this answer



    share|cite|improve this answer










    answered Mar 16 at 17:35









    Dr. Sonnhard GraubnerDr. Sonnhard Graubner

    78.4k42867




    78.4k42867












    • $begingroup$
      WA says the same thing: wolframalpha.com/input/…
      $endgroup$
      – Michael Rozenberg
      Mar 16 at 19:12










    • $begingroup$
      Should i misreaded the problem?
      $endgroup$
      – Dr. Sonnhard Graubner
      Mar 16 at 19:13










    • $begingroup$
      I can't get around proving the inequality with this method. Do you have a different answer?
      $endgroup$
      – Lê Thành Đạt
      Mar 17 at 11:27


















    • $begingroup$
      WA says the same thing: wolframalpha.com/input/…
      $endgroup$
      – Michael Rozenberg
      Mar 16 at 19:12










    • $begingroup$
      Should i misreaded the problem?
      $endgroup$
      – Dr. Sonnhard Graubner
      Mar 16 at 19:13










    • $begingroup$
      I can't get around proving the inequality with this method. Do you have a different answer?
      $endgroup$
      – Lê Thành Đạt
      Mar 17 at 11:27
















    $begingroup$
    WA says the same thing: wolframalpha.com/input/…
    $endgroup$
    – Michael Rozenberg
    Mar 16 at 19:12




    $begingroup$
    WA says the same thing: wolframalpha.com/input/…
    $endgroup$
    – Michael Rozenberg
    Mar 16 at 19:12












    $begingroup$
    Should i misreaded the problem?
    $endgroup$
    – Dr. Sonnhard Graubner
    Mar 16 at 19:13




    $begingroup$
    Should i misreaded the problem?
    $endgroup$
    – Dr. Sonnhard Graubner
    Mar 16 at 19:13












    $begingroup$
    I can't get around proving the inequality with this method. Do you have a different answer?
    $endgroup$
    – Lê Thành Đạt
    Mar 17 at 11:27




    $begingroup$
    I can't get around proving the inequality with this method. Do you have a different answer?
    $endgroup$
    – Lê Thành Đạt
    Mar 17 at 11:27











    1












    $begingroup$

    Another solution.



    Since $$prod_{cyc}(a-1)^2=prod_{cyc}((a-1)(b-1))geq0,$$ we can assume that $(a-1)(b-1)geq0,$ which gives
    $$a+bleq ab+1=frac{1}{c}+1.$$



    Also, we have $$frac{1}{(a+1)^2}+frac{1}{(b+1)^2}-frac{1}{ab+1}=frac{ab(a-b)^2+(ab-1)^2}{(a+1)^2(b+1)^2}geq0$$
    Id est,
    $$sum_{cyc}frac{1}{(a+1)^2}+frac{1}{a+b+c+1}geqfrac{1}{ab+1}+frac{1}{(c+1)^2}+frac{1}{ab+1+c+1}=$$
    $$=frac{1}{frac{1}{c}+1}+frac{1}{(c+1)^2}+frac{1}{frac{1}{c}+1+c+1}=frac{c}{c+1}+frac{1}{(c+1)^2}+frac{c}{(c+1)^2}=1.$$






    share|cite|improve this answer









    $endgroup$


















      1












      $begingroup$

      Another solution.



      Since $$prod_{cyc}(a-1)^2=prod_{cyc}((a-1)(b-1))geq0,$$ we can assume that $(a-1)(b-1)geq0,$ which gives
      $$a+bleq ab+1=frac{1}{c}+1.$$



      Also, we have $$frac{1}{(a+1)^2}+frac{1}{(b+1)^2}-frac{1}{ab+1}=frac{ab(a-b)^2+(ab-1)^2}{(a+1)^2(b+1)^2}geq0$$
      Id est,
      $$sum_{cyc}frac{1}{(a+1)^2}+frac{1}{a+b+c+1}geqfrac{1}{ab+1}+frac{1}{(c+1)^2}+frac{1}{ab+1+c+1}=$$
      $$=frac{1}{frac{1}{c}+1}+frac{1}{(c+1)^2}+frac{1}{frac{1}{c}+1+c+1}=frac{c}{c+1}+frac{1}{(c+1)^2}+frac{c}{(c+1)^2}=1.$$






      share|cite|improve this answer









      $endgroup$
















        1












        1








        1





        $begingroup$

        Another solution.



        Since $$prod_{cyc}(a-1)^2=prod_{cyc}((a-1)(b-1))geq0,$$ we can assume that $(a-1)(b-1)geq0,$ which gives
        $$a+bleq ab+1=frac{1}{c}+1.$$



        Also, we have $$frac{1}{(a+1)^2}+frac{1}{(b+1)^2}-frac{1}{ab+1}=frac{ab(a-b)^2+(ab-1)^2}{(a+1)^2(b+1)^2}geq0$$
        Id est,
        $$sum_{cyc}frac{1}{(a+1)^2}+frac{1}{a+b+c+1}geqfrac{1}{ab+1}+frac{1}{(c+1)^2}+frac{1}{ab+1+c+1}=$$
        $$=frac{1}{frac{1}{c}+1}+frac{1}{(c+1)^2}+frac{1}{frac{1}{c}+1+c+1}=frac{c}{c+1}+frac{1}{(c+1)^2}+frac{c}{(c+1)^2}=1.$$






        share|cite|improve this answer









        $endgroup$



        Another solution.



        Since $$prod_{cyc}(a-1)^2=prod_{cyc}((a-1)(b-1))geq0,$$ we can assume that $(a-1)(b-1)geq0,$ which gives
        $$a+bleq ab+1=frac{1}{c}+1.$$



        Also, we have $$frac{1}{(a+1)^2}+frac{1}{(b+1)^2}-frac{1}{ab+1}=frac{ab(a-b)^2+(ab-1)^2}{(a+1)^2(b+1)^2}geq0$$
        Id est,
        $$sum_{cyc}frac{1}{(a+1)^2}+frac{1}{a+b+c+1}geqfrac{1}{ab+1}+frac{1}{(c+1)^2}+frac{1}{ab+1+c+1}=$$
        $$=frac{1}{frac{1}{c}+1}+frac{1}{(c+1)^2}+frac{1}{frac{1}{c}+1+c+1}=frac{c}{c+1}+frac{1}{(c+1)^2}+frac{c}{(c+1)^2}=1.$$







        share|cite|improve this answer












        share|cite|improve this answer



        share|cite|improve this answer










        answered Mar 17 at 5:03









        Michael RozenbergMichael Rozenberg

        109k1896201




        109k1896201















            Popular posts from this blog

            Magento 2 - Add success message with knockout Planned maintenance scheduled April 23, 2019 at 23:30 UTC (7:30pm US/Eastern) Announcing the arrival of Valued Associate #679: Cesar Manara Unicorn Meta Zoo #1: Why another podcast?Success / Error message on ajax request$.widget is not a function when loading a homepage after add custom jQuery on custom themeHow can bind jQuery to current document in Magento 2 When template load by ajaxRedirect page using plugin in Magento 2Magento 2 - Update quantity and totals of cart page without page reload?Magento 2: Quote data not loaded on knockout checkoutMagento 2 : I need to change add to cart success message after adding product into cart through pluginMagento 2.2.5 How to add additional products to cart from new checkout step?Magento 2 Add error/success message with knockoutCan't validate Post Code on checkout page

            Fil:Tokke komm.svg

            Where did Arya get these scars? Unicorn Meta Zoo #1: Why another podcast? Announcing the arrival of Valued Associate #679: Cesar Manara Favourite questions and answers from the 1st quarter of 2019Why did Arya refuse to end it?Has the pronunciation of Arya Stark's name changed?Has Arya forgiven people?Why did Arya Stark lose her vision?Why can Arya still use the faces?Has the Narrow Sea become narrower?Does Arya Stark know how to make poisons outside of the House of Black and White?Why did Nymeria leave Arya?Why did Arya not kill the Lannister soldiers she encountered in the Riverlands?What is the current canonical age of Sansa, Bran and Arya Stark?